Đến nội dung

Mrnhan

Mrnhan

Đăng ký: 24-07-2012
Offline Đăng nhập: Riêng tư
***--

#683353 Tính đạo hàm Fréchet của: $f(x)=||Ax-b||^2$

Gửi bởi Mrnhan trong 06-06-2017 - 13:38

Em có biết một công thức liên quan đến tính đạo hàm của tích ma trận (product rules) 

$$\nabla \left( U^TV\right)=\nabla(U)V+\nabla(V)U$$

 

Bài toán ban đầu tương đương với 

$$f(x)=\left\|Ax-b\right\|^2=(Ax-b)^T(Ax-b)\Rightarrow f'(x)=2A^T(Ax-b)$$

 

Vì nếu $C= (c_1, c_2, \dots, c_n)^T$ và $x=(x_1, x_2, \dots, x_n)^T$

 

$$\Rightarrow g(x)=C^Tx=c_1x_1+c_2x_2+\dots+c_nx_n$$

$$\Rightarrow \nabla (g(x))=\begin{pmatrix} c_1\\ c_2\\ \vdots\\ c_n \end{pmatrix}=C$$

Vậy $$\nabla(Ax-b)=A^T$$




#667600 $(Au, u)_{L_2(\Omega)}\geq\lambda_0\left|...

Gửi bởi Mrnhan trong 08-01-2017 - 11:28

Cho $A$ là ma trận vuông thực có các trị riêng $\lambda_k>0$ và $\lambda_0=\min\left\{\lambda_k\right\}$.

Chứng minh rằng: $$(A u, u)_{L_2(\Omega)}\geq\lambda_0\left| u\right|^2, \; \forall u$$

trong đó

$$\left(u, v\right)_{L_2(\Omega)}=\int_{\Omega}uvdx$$




#633861 Tìm nghiệm của $\frac{\partial^2u }{\parti...

Gửi bởi Mrnhan trong 18-05-2016 - 11:38

Tìm nghiệm của bài toán:
$$\frac{\partial^2u }{\partial t^2}=\Delta u=\frac{\partial^2 u}{\partial x^2}+\frac{\partial^2 u}{\partial y^2}+\frac{\partial^2 u}{\partial z^2}$$
thỏa mãn điều kiện
$$\left\{\begin{matrix}u(x, y, z, 0)=\varphi(r)\\\frac{\partial u }{\partial t}(x, y, z,0)=\psi(r)\end{matrix}\right. , r=\sqrt{x^2+y^2+z^2}$$



#613646 Tính $\int_0^1 \frac{\ln(1-x^3)}{x}dx...

Gửi bởi Mrnhan trong 08-02-2016 - 15:00

Biết rằng $\int_0^1 \frac{\ln(1+x)}{x}=\frac{\pi^2}{12}$, tính $\int_0^1 \frac{\ln(1-x^3)}{x}dx$

 

Bài giải:

 

Ta có

 

$$t=x^3\Rightarrow I=\int_{0}^{1}\frac{\ln(1-t)}{3t}dt\Rightarrow 3I=\int_{0}^{1}\frac{\ln(1-x)}{x}dx$$ 

 

$$3I+\frac{\pi^2}{12}=\int_{0}^{1}\frac{\ln(1-x)}{x}dx+\int_{0}^{1}\frac{\ln(1+x)}{x}dx=\int_{0}^{1}\frac{\ln(1-x^2)}{x}dx=\int_{0}^{1}\frac{\ln(1-x^2)}{2x^2}d(x^2)=\frac{3I}{2}$$

 

$$\Rightarrow I=-\frac{\pi^2}{18}$$

 

Cách tìm tích phân đầu




#608895 Có thể có bao nhiêu nhóm khác nhau

Gửi bởi Mrnhan trong 14-01-2016 - 11:49

Bài toán:

Chọn 8 học sinh trong tổng số 20 học sinh để nhận học bổng từ 2 nhà tài trợ. Biết rằng những học sinh cùng 1 nhà tài trợ sẽ làm nhóm cùng nhau và mỗi học sinh có thể nhận 2 học bổng (làm việc trong 2 nhóm). Hỏi có thể có bao nhiêu nhóm khác nhau.




#604345 Tìm cực trị $w=x^2+4y^2-3xy+11x-34y+a$

Gửi bởi Mrnhan trong 21-12-2015 - 11:45

 

Câu 1: Tìm cực trị 
$w=x^2+4y^2-3xy+11x-34y+a$
Câu 2: Sử dụng phương pháp nhân tử Lagrange hãy tìm cực trị
$w=axy$
với $x+ay=10$
 
Spoiler

 

 

Lý thuyệt (học cũng được 2 năm rồi :( chả nhớ lắm )

 

Ta có $$\left\{\begin{matrix} A=w''_{x^2}\\B=w''_{xy}\\C=w''_{y^2}\end{matrix}\right. \Rightarrow D=B^2-AC$$

 

Và điểm dừng $M(x_0, y_0)$

$D>0$ thì không có cực trị tại $M.$

$D<0$ thì có cực trị, xét dấu của $A$

$D=0$ thì chưa có kết luận(có thể có thể ko) (chả nhớ xét kiểu gì :D )

 

Bài 1.

 

Tìm điểm dừng

$$\left\{\begin{matrix} w'_x=2x-3y+11=0\\w'_y=8y-3x-34=0\end{matrix}\right.\Rightarrow \left\{\begin{matrix} x=2\\y=5\end{matrix}\right.$$

 

$$\Rightarrow \left\{\begin{matrix} A=w''_{x^2}=2\\B=w''_{xy}=-3\\C=w''_{y^2}=8\end{matrix}\right. \Rightarrow D=B^2-AC=-7, \, A=2>0$$

 

Nên hàm đã cho đạt cực tiểu tại $(2, 5)$ và không có cực đại.

 

Bài 2.

 

Lý thuyết: Tìm cực trị của hàm $f=f(x, y)$ thỏa mãn $g(x, y)=0$. Xét hàm Lagrange

$$F=F(x, y, \lambda)=f(x,y)+\lambda g(x,y)$$

 

Tìm điểm dừng:

$$\left\{\begin{matrix}F'_x=ay+\lambda=0\\F'_y=ax+a\lambda=0\\F'_\lambda=x+ay-10=0 \end{matrix} \right.\Rightarrow \left\{\begin{matrix}a\neq 0\\x=5\\y=-\frac{5}{a}\\\lambda=-5 \end{matrix} \right.$$

 

$$\Rightarrow \Delta=-\begin{vmatrix} 0&&g'_x&&g'_y\\g'_x&&F''_{x^2}&&F''_{xy}\\g'_y&&F''_{xy}&&F''_{y^2}\end{vmatrix}=a^4$$

 

Hàm đã cho có cực tiểu tại $\left ( 5,\,\frac{5}{a} \right )$.

 

Có tham khảo tại đây.




#603724 Tổng kết 5 năm hoạt động của Chương trình trọng điểm quốc gia phát triển Toán...

Gửi bởi Mrnhan trong 18-12-2015 - 11:02

Có thành viên nào của VMF tham gia cái này không? 

Đến gặp mặt :D (chụp ảnh cũng vui)

File gửi kèm  maths.jpg   102.09K   198 Số lần tải




#602724 Tìm các hệ số của $y = a_0x + a_1x^3 + a_2x^5 + ... + a_nx^{2n + 1} + .....

Gửi bởi Mrnhan trong 12-12-2015 - 08:55

Cho $y = a_0x + a_1x^3 + a_2x^5 + ... + a_nx^{2n + 1} + ...$ Thỏa mãn $\left (1 - x^2 \right )y' - xy = 1, x \in \left (-1; 1 \right )$
Tìm các hệ số $a_0, a_1, a_2, ..., a_n$

Học sinh giỏi Bắc Ninh $2009$

 

 

Từ đề bài, ta có $$y(0) = 0, y'(0)=1$$

 

Đặt $$y = \frac{b_1}{1!}x+\frac{b_2}{2!}x^2+...+\frac{b_n}{n!}x^n+...$$

 

$$\Rightarrow b_n = y^{(n)}(0)$$

 

Ta cần tìm các  hệ số của pt (dựa vào giả thiết):

$$(1-x^2)y^{(n+2)}=k_{n}xy^{(n+1)}+t_{n}y^{(n)}$$

$$b_{n+2}=t_nb_n$$

 

Theo giả thiết, ta có

 

$$\left (1 - x^2 \right )y' = xy + 1 \Rightarrow (1-x^2)y''=3xy'+y$$

 

$$\Rightarrow (1-x^2)y'''= 5xy''+4y'$$

 

$$\Rightarrow (1-x^2)y^{(4)}= 7xy^{(3)}+9y''$$

$$....$$

 
Đoán: $k_n = 2n+3, \,\, t_{n}=k_{n-1}+t_{n-1}=t_{n-1}+2n+1, t_0=1$ (quy nạp lại để chứng minh :D )
 
Dễ dàng suy ra $$t_{n}=(n+1)^2\to b_{n+2}=(n+1)^2b_n,\,\, b_0=0, \,\,b_1=1$$
 
*************************************
Làm tiếp:
 
$$\Rightarrow \left\{\begin{matrix} b_{2n}=0\\b_1=1\\b_{2n+1}=4n^2b_{2n-1}=...=4^n(n!)^2b_1=4^n(n!)^2\end{matrix}\right.$$
 
$$\Rightarrow a_n=\frac{b_{2n+1}}{(2n+1)!}=\frac{4^n(n!)^2}{(2n+1)!}$$
 
Giải phương trình:
 
$$(1-x^2)y'-xy=1\Leftrightarrow \sqrt{1-x^2}y'-\frac{xy}{\sqrt{1-x^2}}=\frac{1}{\sqrt{1-x^2}}$$
 
$$\left ( y\sqrt{1-x^2} \right )'=\frac{1}{\sqrt{1-x^2}}\Rightarrow y=\frac{\arcsin(x)+C}{\sqrt{1-x^2}}=\frac{\arcsin(x)}{\sqrt{1-x^2}}$$
 
So sánh kết quả với khai triển: wolframalpha



#593418 $\left| {{a_{n + 1}} - {a_n}...

Gửi bởi Mrnhan trong 12-10-2015 - 12:56

Mình xin nhờ các anh, chị, các bạn hướng dẫn hộ mình bài này.

Chứng minh dãy số ${\left\{ {{a_n}} \right\}_n}$ thỏa mãn $\left| {{a_{n + 1}} - {a_n}} \right| < {\left( {\frac{{2014}}{{2015}}} \right)^n},\forall n \in {N^*}$ là dãy Cauchy.

Mình xin cám ơn và chúc mọi người một đêm thật ngon giấc. :)

 

 

$\forall \epsilon >0 ,\,\forall q, p \in \mathbb{N},\, p>q> \left \lceil \frac{2015}{2014}\ln\frac{\epsilon }{2015} \right \rceil$, ta có

 

$$\left | a_p-a_q \right |\leq \left | a_p-a_{p-1} \right |+...+\left | a_{q+1}-a_q \right |<\left ( \frac{2014}{2015} \right )^{p-1}+...+\left ( \frac{2014}{2015} \right )^q $$

 

$$= \left ( \frac{2014}{2015} \right )^q\left [ \left ( \frac{2014}{2015} \right )^{p-q-1}+...+1 \right ]=\left ( \frac{2014}{2015} \right )^q \times \frac{1-\left ( \frac{2014}{2015} \right )^{p-q}}{1-\frac{2014}{2015}}<2015 \times \left ( \frac{2014}{2015} \right )^q$$

 

$$\Rightarrow \left | a_p-a_q \right |<2015 \times \left ( \frac{2014}{2015} \right )^q < \epsilon,\,\, \boxed{\text{đpcm}}$$




#582205 Tính $\int_{0}^{+\infty }\frac{x...

Gửi bởi Mrnhan trong 16-08-2015 - 06:43

Tính tích phân

 

$$\int_{0}^{+\infty }\frac{x^{2}}{x^{4}-x^{2}+1}dx$$

 

Ta có

 

$$I=\int_{0}^{\infty} \frac{x^2}{x^4-x^2+1}dx=\frac{1}{2}\int_{-\infty}^{\infty}\frac{x^2}{x^4-x^2+1}dx$$

 

$$=\pi i Re\left \{ \frac{x^2}{x^4-x^2+1}, \, x=\frac{\sqrt{3}}{2}+\frac{1}{2}i \right \}+\pi i Re\left \{ \frac{x^2}{x^4-x^2+1}, \, x=-\frac{\sqrt{3}}{2}+\frac{1}{2}i \right \}$$

 

$$=\pi i \left ( \frac{\sqrt{3}}{12}-\frac{1}{4}i \right )+\pi i\left ( -\frac{\sqrt{3}}{12}-\frac{1}{4}i \right )=\frac{\pi}{2}$$




#569625 THƯ MỜI HỌP MẶT 5/7/2015 tại Tp. Hồ Chí Minh

Gửi bởi Mrnhan trong 03-07-2015 - 11:10

Khi nào mới tổ chức ở Hà Nội thế BQT :)




#568599 $ \sqrt[3]{25x(2x^2+9)} \geq 4x+\dfrac{3}{x} $

Gửi bởi Mrnhan trong 28-06-2015 - 07:20

 

Giải bất phương trình:

$ \sqrt[3]{25x(2x^2+9)} \geq 4x+\dfrac{3}{x} $

 

 

$$\sqrt[3]{25x(2x^2+9)} \geq 4x+\dfrac{3}{x}\Leftrightarrow x\left ( \sqrt[3]{25\left ( 2+\frac{9}{x^2} \right )}-4-\frac{3}{x^2} \right )\geq 0$$

 

$$\Leftrightarrow \frac{-x(\frac{3}{x^2}-1)^2(\frac{3}{x^2}+14)}{\left ( \sqrt[3]{25\left ( 2+\frac{9}{x^2} \right )} \right )^2-\left ( 4+\frac{3}{x^2} \right )\sqrt[3]{25\left ( 2+\frac{9}{x^2} \right )} +\left ( 4+\frac{3}{x^2} \right )^2 }\geq 0$$

 

$$\Leftrightarrow x < 0$$

 

Lớn mặt rồi làm mấy này thế Trang tròn :) Ở YHN cũng học toán này à??




#567566 Xin tài liệu Học Máy, TTNT

Gửi bởi Mrnhan trong 23-06-2015 - 07:14

Ai có tài liệu Học Máy (Machine learning) hoặc Trí Tuệ Nhân Tạo (AI) bằng Tiếng Việt không, dốt tiếng anh nên đọc không hiểu :( 


  • Nxb yêu thích


#564787 $\sum_{k=1}^{\infty}arc\tan\left...

Gửi bởi Mrnhan trong 10-06-2015 - 13:46

 
Em nhớ câu 1 là $\arctan\frac{1}{k^2}$, lúc đó làm mãi ko ra :D
 
1. Nhận xét
 
$$\arctan\frac{2}{k^2}=\arctan\frac{(k+1)-(k-1)}{1+(k+1)(k-1)}=\arctan(k+1)-\arctan(k-1)$$
$$S(1)=\sum_{k=1}^{\infty}\arctan\frac{2}{k^2}=\lim_{k\to \infty}\left(-\arctan1+\arctan k + \arctan(k+1)\right)=\frac{3\pi}{4}$$
 
2. Phân kỳ :)
 
 
Tổng quát:
 

 

$$S(a)=\sum_{k=1}^{\infty} \arctan\frac{2a^2}{k^2},\, a>0$$

 

 

Đặt 
 
$$F(a)=\sum_{k=-\infty}^{\infty}\arctan\frac{2a^2}{k^2}=2S(a)+\frac{\pi}{2}$$
 
$$\Rightarrow F'(a)=\sum_{k=-\infty}^{\infty}\frac{4ak^2}{k^4+4a^4}$$
 
$$=-\pi Res\left\{\frac{4ak^2\cot(\pi k)}{k^4+a^4}, k=a(i+1)\right\}-\pi Res\left\{\frac{4ak^2\cot(\pi k)}{k^4+a^4}, k=a(i-1)\right\}$$
 
$$=-\pi \left(\frac{\cot\left(\pi a(i+1)\right)}{i+1}+\frac{\cot\left(\pi a(i-1)\right)}{i-1}\right)$$
 
$$\Rightarrow F(a)=-\left(\frac{\ln\sin\left(\pi a(i+1)\right)}{(i+1)^2}+\frac{\ln\sin\left(\pi a(i-1)\right)}{(i-1)^2}\right)+C$$
 
$$=\frac{i}{2}\ln\frac{\sin\left(\pi a(i+1)\right)}{\sin\left(\pi a(i-1)\right)}+C$$
 
$$\Rightarrow F(1)=C=2\pi$$
 
$$\Rightarrow S(a)=\frac{i}{4}\ln\frac{\sin\left(\pi a(i+1)\right)}{\sin\left(\pi a(i-1)\right)}+\frac{3\pi}{4}$$
 
 



#563158 Tính $\int_{0}^{+\infty }\frac{l...

Gửi bởi Mrnhan trong 03-06-2015 - 06:33

Tính

                                            $\int_{0}^{+\infty }\frac{lnxdx}{1-x^{2}}$

 

$$I=\int_{0}^{\infty}\frac{\ln x}{1-x^2}dx=\int_{0}^{1}\frac{\ln x}{1-x^2}dx+\int_{1}^{\infty}\frac{\ln x}{1-x^2}dx$$
$$=\int_{0}^{1}\frac{\ln x}{1-x^2}dx+\int_{0}^{1}\frac{\ln x}{1-x^2}dx$$
$$=2\int_{0}^{1}\frac{\ln x}{1-x^2}dx$$
$$=\int_{0}^{1}\frac{\ln x}{1-x}+\int_{0}^{1}\frac{\ln x}{1+x}dd$$